Đến nội dung

VNSTaipro nội dung

Có 319 mục bởi VNSTaipro (Tìm giới hạn từ 19-04-2020)



Sắp theo                Sắp xếp  

#494334 $\sum a^{3}(b^{2}+c^{2})\leq 2...

Đã gửi bởi VNSTaipro on 21-04-2014 - 11:32 trong Bất đẳng thức - Cực trị

Với $a,b,c$ là các số không âm thỏa $a+b+c=2$. Chứng minh:

$\sum a^{3}(b^{2}+c^{2})\leq 2$




#492832 Giải bất phương trình vô tỉ

Đã gửi bởi VNSTaipro on 14-04-2014 - 10:37 trong Phương trình - hệ phương trình - bất phương trình

Bài này mình giải k pít đúng k, m.n xem sai chỗ nào nhé

 

$\sqrt{2x+\sqrt{x^{2}+1}}>x+1 \Leftrightarrow \begin{bmatrix} \left\{\begin{matrix} x+1<0\\ 2x+\sqrt{x^{2}+1}\geq 0\end{matrix}\right.\\ \left\{\begin{matrix} x+1\geq 0\\ 2x+\sqrt{x^{2}+1}>x^{2}+2x+1\end{matrix}\right.\end{bmatrix} \Leftrightarrow \begin{bmatrix} \left\{\begin{matrix} x<-1\\ \sqrt{x^{2}+1}\geq -2x\end{matrix}\right.\\ \left\{\begin{matrix} x\geq -1\\ \sqrt{x^{2}+1}>x^{2}+1\end{matrix}\right.\end{bmatrix} \Leftrightarrow \begin{bmatrix} \left\{\begin{matrix} x<-1\\ 3x^{2}\leq 1\end{matrix}\right.\\ \left\{\begin{matrix} x\geq -1\\ \sqrt{x^{2}+1}-1<0\end{matrix}\right.\end{bmatrix}\Leftrightarrow \begin{bmatrix} \left\{\begin{matrix} x<-1\\ \frac{-1}{\sqrt{3}}\leq x\leq \frac{1}{\sqrt{3}}\end{matrix}\right.\\ \left\{\begin{matrix} x\geq -1\\ x<0\end{matrix}\right.\end{bmatrix}\Leftrightarrow \begin{bmatrix} vô lí\\ -1\leq x<0\end{bmatrix}\Leftrightarrow -1\leq x<0$

 

Nhưng thay x = -1 thì căn thức k xác định, vậy thì lúc giải TH ở nhánh hoặc thứ 2 vẫn phải tìm đk cho căn thức, nhưng theo phương pháp giải thì chỉ cần x + 1 >= 0

$f(x)>g(x)\Leftrightarrow \begin{bmatrix} \left\{\begin{matrix} g(x)<0\\ f(x)\geq 0\end{matrix}\right.\\ \left\{\begin{matrix} g(x)\geq 0\\ f(x)>g(x)^{2}\end{matrix}\right.\end{bmatrix}$

Thế thì sai chỗ nào vậy? Còn cách làm nào khác k

Lúc đầu cần tìm điều kiện để căn có nghĩa đã bạn, là $x\geq 0$ hoặc $\frac{-1}{\sqrt{3}}\leq x\leq \frac{1}{\sqrt{3}}$




#482554 $\left\{\begin{matrix} x^{3}=2y-...

Đã gửi bởi VNSTaipro on 11-02-2014 - 18:26 trong Phương trình - hệ phương trình - bất phương trình

c2:
 

hoặc bạn xét hàm số: $f_{(t)}=t^3-2t+1$

đây là hàm đồng biến nên ta suy ra $x=y=z$

 

đến đây bạn thế vào là được: $x=y=z=\frac{\pm \sqrt{5}\pm 1}{2} ,Vx=y=z=1$

 

đây là một hướng khác để được $x=y=z$

Hàm này chưa chắc đồng biến :))




#481894 P= $\frac{a^3}{\sqrt{b^2+3}}...

Đã gửi bởi VNSTaipro on 08-02-2014 - 11:19 trong Bất đẳng thức và cực trị

bất dẳng thức sao kì vậy bạn, bạn viết rõ dùm mình đi, mình làm mãi không ra đượctừ bước 2 sang 3

Viết lộn đó bạn. Dùng AM-GM dưới mẫu là ok




#481893 Tìm gtnn, gtln của $P=\sum \frac{x+y}{1+z}...

Đã gửi bởi VNSTaipro on 08-02-2014 - 11:12 trong Bất đẳng thức và cực trị

Bạn giải thích kĩ hơn phần biến đổi đầu đk k? Mình chưa hiểu lắm :(

Nhân $(x+y)$ cả tử và mẫu rồi dùng trực tiếp Cauchy Schwarz




#481396 Tìm min, max của $P=\sum \frac{x+y}{1+z}...

Đã gửi bởi VNSTaipro on 06-02-2014 - 15:43 trong Bất đẳng thức và cực trị

Cho x,y,z là các số thực nằm trong đoạn $\left [ \frac{1}{2};1 \right ]$. Tìm gtnn, gtln của biểu thức:

$P=\frac{x+y}{1+z}+\frac{z+y}{1+x}+\frac{x+z}{1+y}$

Có ở đây nè bạn http://diendantoanho...-psum-fracxy1z/




#478998 tìm min của bt P=$\frac{1}{a^2+1}+\frac...

Đã gửi bởi VNSTaipro on 25-01-2014 - 19:22 trong Bất đẳng thức và cực trị

Nếu đưa về lượng giác thf phả a+b+c=abc chứ bạn đâu như bạn nghĩ

$a+b+c=abc$ hay $a+b+abc=c$ đều được bạn :D




#478847 Tìm GTNN của: $A=-12ab-4ac-34bc$

Đã gửi bởi VNSTaipro on 24-01-2014 - 20:56 trong Bất đẳng thức - Cực trị

Cho a, b, c không âm thỏa mãn; $a+b+c=4$. Tìm GTNN của: $A=-12ab-4ac-34bc$

Bạn xem lại đề giùm mình cái,hệ số như vậy không làm được




#478845 $\sum \frac{a}{b}\geqslant \frac...

Đã gửi bởi VNSTaipro on 24-01-2014 - 20:51 trong Bất đẳng thức - Cực trị

CMR $\sum \frac{a}{b}\geqslant \frac{a+b}{b+c}+\frac{b+c}{a+b}+1$

BDT $\Leftrightarrow (\frac{a}{b}-\frac{a}{b+c})+(\frac{b}{c}-\frac{b}{b+c})+(\frac{c}{a}-\frac{c}{a+b})\geq \frac{b}{a+b}+1$

$\Leftrightarrow \frac{ac}{b(b+c)}+\frac{b^2}{c(b+c)}+\frac{bc}{a(a+b)}\geq \frac{a+2b}{a+b}$

$VT= \frac{1}{b+c}(\frac{ac}{b}+\frac{b^2}{c})+\frac{bc}{a(a+b)}$

Mà $(\frac{ac}{b}+\frac{b^2}{c})(abc+a^2c)\geq (ac+ab)^{2}$

$\Rightarrow (\frac{ac}{b}+\frac{b^2}{c})\geq \frac{a^2(b+c)^2}{ac(a+b)}$

$\Rightarrow VT\geq \frac{a(b+c)}{c(a+b)}+\frac{bc}{a(a+b)}\geq \frac{a+2b}{a+b}$

$\Leftrightarrow \frac{a(b+c)}{c}+\frac{bc}{a}\geq a+2b$

$\Leftrightarrow a^2b+bc^2\geq 2abc$ (Đúng)



#478829 tìm min của bt P=$\frac{1}{a^2+1}+\frac...

Đã gửi bởi VNSTaipro on 24-01-2014 - 20:28 trong Bất đẳng thức và cực trị

Cho ba số dương a, b, c thỏa mãn a+b+ab=c. Tìm GTNN của biểu thức 

P=$\frac{1}{a^2+1}+\frac{2}{b^2+1}-\frac{3}{c^2+1}$

Bạn xem lại giả thiết đi bạn, phải là $a+b+abc=c$ để đưa về lượng giác




#478826 Giải hệ phương trình

Đã gửi bởi VNSTaipro on 24-01-2014 - 20:23 trong Phương trình - Hệ phương trình - Bất phương trình

Giải hệ phương trình

 $\left\{\begin{matrix}
&x^2\left(y+3\right)=4\left(2-y\right) \\
& y^2\left(z+3\right)=4\left(2-z\right) \\
& z^2\left(x+3\right)=4\left(2-x\right)
\end{matrix}\right.$

Ai giúp mình với!

Bài này có điều kiện $a,b,c>0$ không bạn




#478824 Tìm gtnn, gtln của $P=\sum \frac{x+y}{1+z}...

Đã gửi bởi VNSTaipro on 24-01-2014 - 20:19 trong Bất đẳng thức và cực trị

$MAX$ mới nghĩ ra

$P=\sum \frac{x+y}{1+z}=\sum \frac{x}{1+z}+\sum \frac{y}{1+z}$

$\leq \sum \frac{x}{x+z}+\sum \frac{y}{y+z}=3$
(Vì $x,y,z\leq 1$)



#478736 Tìm gtnn, gtln của $P=\sum \frac{x+y}{1+z}...

Đã gửi bởi VNSTaipro on 24-01-2014 - 10:13 trong Bất đẳng thức và cực trị

Cho x,y,z là các số thực nằm trong đoạn $\left [ \frac{1}{2};1 \right ]$. Tìm gtnn, gtln của biểu thức:

$P=\frac{x+y}{1+z}+\frac{z+y}{1+x}+\frac{x+z}{1+y}$

$P= \sum \frac{x+y}{1+z}=\sum \frac{(x+y)^2}{(x+y)+z(x+y)}$

$\geq \frac{4(x+y+z)^2}{2\sum x+2\sum xy}$
$\geq \frac{2(\sum x)^2}{\sum x+\frac{1}{3}(\sum x)^2}$
$=\frac{6\sum x}{3+\sum x}$
Vì $\frac{3}{2}\leq \sum x\leq 3$ nên $MIN P=2$ khi $x=y=z=\frac{1}{2}$



#478613 Chứng minh rằng: $a^2b+c^2a+2abc\leq 20$

Đã gửi bởi VNSTaipro on 23-01-2014 - 17:39 trong Bất đẳng thức và cực trị

Cho $a, b, c\geq 0$ thoả mãn $a+b+c=5$. Chứng minh rằng:

$$a^2b+c^2a+2abc\leq 20$$

$VT=a^2b+c^2a+2abc+bc^2-bc^2$

$=b(a+c)^2+c^2(a-b)$
$TH1$ $a\leq b \Rightarrow VT\leq b(a+c)^2=\frac{1}{2}2b(a+c)(a+c)\leq \frac{1}{2}(\frac{2(a+b+c)}{3})^{3}<20$ 
$TH2$ $a\geq b\geq 0 \Rightarrow VT=b(a+c)^2+c^2(a-b)=b(5-b)^{2}+4.\frac{c}{2}.\frac{c}{2}(a-b)$
$\leq b(5-b)^2+4(\frac{c+a-b}{3})^{3}$
$=b(5-b)^2+4(\frac{5-2b}{3})^{3}$
Đến đây xét hàm theo $b$ với $0\leq b\leq \frac{5}{2}$ thì tìm được $GTLN$ khi $b=1$ thì $VT= 20$
Vậy dấu $=$ xảy ra khi $a=2, b=1, c=2 $



#478612 $\sum \frac{\left ( a+b-c \right )^{2...

Đã gửi bởi VNSTaipro on 23-01-2014 - 17:28 trong Bất đẳng thức và cực trị



Lâu lắm rồi không đưa người yêu lên...

 

Bài toán: Cho $a, b, c$ là các số thực dương. CM

 

$\frac{\left ( a+b-c \right )^{2}}{a^{2}+b^{2}+c^{2}+2ab}+\frac{\left ( b+c-a \right )^{2}}{a^{2}+b^{2}+c^{2}+2bc}+\frac{\left ( c+a-b \right )^{2}}{a^{2}+b^{2}+c^{2}+2ca}\geq \frac{3}{5}$

 

-------------------------------------------------------------------------------------------------

 

                   Đề thi thử đại học lần 1 khối A, $A_{1}$ tỉnh Vĩnh Phúc.

                    

 

                               

$VT$ $=\sum \frac{a^2+b^2+c^2+2ab-2bc-2ca}{a^2+b^2+c^2+2ab}$

$=\sum (1-\frac{2c(a+b)}{(a+b)^{2}+c^{2}})$
$=\sum (1-\frac{2c(a+b)}{\frac{3}{4}(a+b)^2+[\frac{1}{4}(a+b)^2+c^2]})$
$\geq \sum(1-\frac{2c(a+b)}{\frac{3}{4}(a+b)^2+c(a+b)})$
$=\sum (1-\frac{8c}{3a+3b+4c})$
Vậy cần chứng minh $\sum (1-\frac{8c}{3a+3b+4c})\geq \frac{3}{5}$
$\Leftrightarrow \sum (2-\frac{8c}{3a+3b+4c})\geq \frac{3}{5}+3=\frac{18}{5}$
$\Leftrightarrow \sum \frac{6a+6b}{3a+3b+4c}\geq \frac{18}{5}$
$\Leftrightarrow \sum \frac{a+b}{3a+3b+4c}\geq \frac{3}{5}$
BĐT này đúng vì $\sum \frac{a+b}{3a+3b+4c}= \sum \frac{(a+b)^{2}}{3(a+b)^2+c(a+b)}$
$\geq \frac{4(a+b+c)^2}{\sum [ 3(a+b)^2+c(a+b)]}\geq \frac{3}{5}$
$\Leftrightarrow 2(a^2+b^2+c^2)\geq 2(ab+bc+ca)$ Đúng



#478572 P=2ab+ 3bc+3ca+$\frac{6}{a+b+c}$

Đã gửi bởi VNSTaipro on 23-01-2014 - 10:33 trong Bất đẳng thức và cực trị

Cho các số thực a, b, c sao cho a ≥ 0, b ≥ 0, 0 ≤ c ≤ 1 và a2 + b2 + c2 = 3. Tìm giá trị lớn nhất và giá trị nhỏ nhất của biểu thức sau:

P=2ab+ 3bc+3ca+$\frac{6}{a+b+c}$

Đề là $2ab+3bc+3ca$ hay là $3(ab+bc+ca)$ vậy bạn




#478173 $\sum \sqrt{a+(b-c)^{2}}\geq \sq...

Đã gửi bởi VNSTaipro on 20-01-2014 - 08:17 trong Bất đẳng thức - Cực trị

Với $a,b,c>0$ $a+b+c=1$. Chứng minh:

$\sum \sqrt{a+(b-c)^{2}}\geq \sqrt{3}$




#477577 $P=a+b+c-\frac{1}{2}(\sum \sqrt[3]...

Đã gửi bởi VNSTaipro on 16-01-2014 - 18:54 trong Bất đẳng thức - Cực trị

Với $a,b,c>0$ thoả $ab+bc+ca=3abc$. Tìm MIN của:

$P=a+b+c-\frac{1}{2}(\sum \sqrt[3]{\frac{a^{3}+b^{3}}{2}})$




#469444 $\frac{\sum a^{2}}{\sum ab}...

Đã gửi bởi VNSTaipro on 07-12-2013 - 16:44 trong Bất đẳng thức - Cực trị

Với $a,b,c$ là các số không âm. Chứng minh:

$\frac{\sum a^{2}}{\sum ab}+\frac{8abc}{\prod (a+b)}\geq 2$




#469264 GTNN của $\sum \frac{a^{2}}{b+c+d...

Đã gửi bởi VNSTaipro on 06-12-2013 - 17:51 trong Bất đẳng thức - Cực trị

Cho $a,b,c,d,e$ là các số thực thỏa $a^{2}+b^{2}+c^{2}+d^{2}+e^{2}=1$

Tìm Min của $\frac{a^{2}}{b+c+d}+\frac{b^{2}}{c+d+e}+\frac{c^{2}}{d+e+a}+\frac{d^{2}}{e+a+b}+\frac{e^{2}}{a+b+c}$




#462751 $\sum \frac{1}{a^{2}+b^{2}+...

Đã gửi bởi VNSTaipro on 07-11-2013 - 20:55 trong Bất đẳng thức và cực trị

Cho $a,b,c$ dương thỏa $a+b+c=3$. Chứng minh

$\sum \frac{1}{a^{2}+b^{2}+2}\leq \frac{3}{4}$




#462238 $\frac{1}{1+ab}+\frac{1}{1+...

Đã gửi bởi VNSTaipro on 05-11-2013 - 16:14 trong Bất đẳng thức và cực trị

Với $a,b,c$ dương; $a+b+c=3$.Chứng minh:

$\frac{1}{1+ab}+\frac{1}{1+bc}+\frac{1}{1+ca}\geq \frac{9}{2(\sqrt{a}+\sqrt{b}+\sqrt{c})}$




#457961 $(1+\frac{1}{abc})(a+b+c)\geq 3+\frac...

Đã gửi bởi VNSTaipro on 16-10-2013 - 19:30 trong Bất đẳng thức và cực trị

Đặt $a=\frac{x}{y},b=\frac{y}{z},c=\frac{z}{x}$

Ta có :$a+b+c=\frac{x}{y}+\frac{y}{z}+\frac{z}{x}\geq 3\sqrt[3]{\frac{xyz}{xyz}}=3$

Mà :$\frac{x}{y}+\frac{y}{z}+\frac{z}{x}\geq \frac{y}{x}+\frac{z}{y}+\frac{x}{z}< = > x^2z+y^2x+z^2y\geq y^2z+x^2y+xz^2< = > xz(x-z)+y^2(x-z)-y(x-z)(x+z)\geq 0< = > (x-z)(xz+y^2-xy-yz)\geq 0< = > (x-z)(x(z-y)-y(z-y))\geq 0< = > (x-z)(x-y)(z-y)\geq 0$

Nếu đặt như vậy là tự cho $abc=1$ rồi bạn




#457119 $(1+\frac{1}{abc})(a+b+c)\geq 3+\frac...

Đã gửi bởi VNSTaipro on 12-10-2013 - 16:45 trong Bất đẳng thức và cực trị

Cho $0<a,b,c\leq 1$. Chứng minh:

$(1+\frac{1}{abc})(a+b+c)\geq 3+\frac{1}{a}+\frac{1}{b}+\frac{1}{c}$

 




#453735 $\frac{2a+c}{1+ac}+\frac{2b+c}...

Đã gửi bởi VNSTaipro on 28-09-2013 - 21:34 trong Bất đẳng thức và cực trị

Cho $a,b,c$ là các số không âm thỏa $a^{2}+b^{2}+c^{2}=1$. Tìm GTLN của

$\frac{2a+c}{1+ac}+\frac{2b+c}{1+bc}+\frac{a+b+c}{1\sqrt{2}abc}$